Let $n$ be a positive integer. Consider a triangular array of nonnegative integers as follows: \[ \begin{array}{rccccccccc} \text{Row } 1: &&&&& a_{0,1} &&&& \smallskip\\ \text{Row } 2: &&&& a_{0,2} && a_{1,2} &&& \smallskip\\ &&& \vdots && \vdots && \vdots && \smallskip\\ \text{Row } n-1: && a_{0,n-1} && a_{1,n-1} && \cdots && a_{n-2,n-1} & \smallskip\\ \text{Row } n: & a_{0,n} && a_{1,n} && a_{2,n} && \cdots && a_{n-1,n} \end{array} \] Call such a triangular array stable if for every $0 \le i < j < k \le n$ we have \[ a_{i,j} + a_{j,k} \le a_{i,k} \le a_{i,j} + a_{j,k} + 1. \] For $s_1, \ldots s_n$ any nondecreasing sequence of nonnegative integers, prove that there exists a unique stable triangular array such that the sum of all of the entries in row $k$ is equal to $s_k$.
Problem
Source: USA TSTST 2012, Problem 8
Tags: induction, linear algebra, matrix, ceiling function, combinatorics unsolved, combinatorics
20.07.2012 09:13
we use induction for this problem .suppose that we are done for $n$ then we prove we are done for $n+1$.we have a unique stable triangular array by order $n$ and last row .we attempt to pick last row unique . let $f(a_{j,n})=-a_{n,n+1}+a_{j,n+1}-a_{j,n}$ we know its $1$ or $0$and $g(a_{i,j})=a_{i,n}-a_{j,n}-a_{i,j}$ .and easy to see we must have $-(n-1)a_{n,n+1}+\sum _{i=0}^{n-1}f(a_{i,n})=s_{n+1}-s_{n}$ and $\sum _{i=0}^{n-1}f(a_{i,n})\leq n$ it means $a_{n,n+1}$ is unique and $\sum _{i=0}^{n-1}f(a_{i,n})$ is unique too.but suppose $\sum _{i=0}^{n-1}f(a_{i,n})=t$ then we know that $\sum g(a_{i,j})=t$ so we pick $f(a_{i,n})=g(a_{i,j})$ and $f(a_{j,n})=0$if and only if $g(a_{i,j})=1$. we are done because we know $-a_{n,n+1}+a_{j,k+1}=a_{j,k}+f(a_{j,k})$ and $-a_{n,n+1}+a_{i,k+1}=a_{i,k}+f(a_{i,k})$ ($i\leq j$).so $(a_{i,n+1}-a_{n,n+1})-(a_{j,n+1}-a_{n,n+1})+f(a_{j,n})+f(a_{i,n})=a_{i,n}-a_{j,n}=1+a_{i,j}$ or $(a_ {i,n+1}-a_{n,n+1})-(a_{j,n+1}-a_{n,n+1})+f(a_{j,n})+f(a_{i,n})=a_{i,n}-a_{j,n}=a_{i,j}$
20.07.2012 17:22
shekast-istadegi wrote: suppose $\sum _{i=0}^{n-1}f(a_{i,n})=t$ then we let $f(a_{1,n})=1,f(a_{2,n})=1,...,f(a_{t},n)=1$ and $f(a_{v,n})=0$ for all $ v> t$ . This construction does not always work. The unique triangle for $n = 3$ and the row sums $0, 1, 2$ is 0 10 110 This has $f(a_{1,3}) = 0$ and $f(a_{2,3}) = 1$. I am assuming your definition of $f$ was intended to be $a_{j,n+1} - a_{n,n+1} - a_{j,n}$ since the current one makes no sense at all.
20.07.2012 18:10
I suppose I'll go ahead and post the original solution I had when I proposed the problem.
Here's a second method, based on some ideas of solutions given during the contest.
23.07.2012 15:37
MellowMelon wrote: shekast-istadegi wrote: suppose $\sum _{i=0}^{n-1}f(a_{i,n})=t$ then we let $f(a_{1,n})=1,f(a_{2,n})=1,...,f(a_{t},n)=1$ and $f(a_{v,n})=0$ for all $ v> t$ . This construction does not always work. The unique triangle for $n = 3$ and the row sums $0, 1, 2$ is 0 10 110 This has $f(a_{1,3}) = 0$ and $f(a_{2,3}) = 1$. I am assuming your definition of $f$ was intended to be $a_{j,n+1} - a_{n,n+1} - a_{j,n}$ since the current one makes no sense at all. so i edit it.
15.08.2015 23:51
23.08.2018 21:02
30.11.2020 22:04
We induct on $n$, with $n=1$ and $n=2$ being easy to solve. Observe that when we have a $n$-triangular array which is stable, if we delete the last row, the reamaining triangular array is also stable. Thus, for any nondecreasing sequence $s_1,s_2,...,s_n$ of nonnegative integers, we can ensure the uniquiness of the $a_{i,j}$ for $0 \leq i < j \leq n-1$. Now, we prove that $a_{0,n}$ is unique: Note that from the array's stability, $$a_{0,i}+a_{i,n} \leq a_{0,n} \leq a_{0,i}+a_{i,n}+1$$for all $i=0,1,...,n-1$. Summing everything, we have that $$a_{0,1}+a_{0,2}+...+a_{0,n-1}+s_n-a_{0,n} \leq (n-1)a_{0,n} \leq a_{0,1}+a_{0,2}+...+a_{0,n-1}+s_n-a_{0,n}+ n-1$$$\implies a_{0,1}+a_{0,2}+...+a_{0,n-1}+s_n \leq na_{0,n} \leq a_{0,1}+a_{0,2}+...+a_{0,n-1}+s_n+n-1$, and then $a_{0,n}$ is unique, say by Division Algorithm. Then, we use induction again to show that $a_{i,n}$ is unique. The basecase is done. Suppose that we have already proved that $a_{0,n},a_{1,n},...,a_{i-1,n}$ are unique. Hence, by the arrays's stability, $$a_{i,j}+a_{j,n} \leq a_{i,n} \leq a_{i,j}+a_{j,n}+1$$for all $j=i+1,i+2,...,n-1$. Summing everything, we have that $$a_{i,i+1}+a_{i,i+2}+...+a_{i,n-1}+(s_n-a_{0,n}-a_{1,n}-...-a_{i-1,n}) \leq (n-i-1)a_{i,n} \leq a_{i,i+1}+a_{i,i+2}+...+a_{i,n-1}+(s_n-a_{0,n}-a_{1,n}-...-a_{i-1,n})+n-i-2$$so we are done by the Division Algorithm. Thus, $a_{0,n},a_{1,n},...,a_{n-1,n}$ are unique when $s_1,s_2,...,s_n$ are given and the array is stable, as desired. $\blacksquare$